Confirm Physics Textbook Error: Trampolinist Spring Constant

In summary, the conversation was about finding the spring constant for a trampolinist bouncing on a mat. The question required finding the period and then using it to calculate the spring constant. The book answer of 3.9kN/m seems to be correct, while the initial answer of 3.5kN/m was incorrect due to a mistake in the calculation. The conversation ended with a humorous comment about "bashing my own head with textbook".
  • #1
Ukitake Jyuushirou
124
0
hello

i think i may have found an error in one of the answer in my physics textbook but i want to confirm it with the physics experts here

Question:

A trampolinist of 55kg bounces in the middle of a trampoline mat. She finds that she bounces 80 times in a min. What is the spring constant?

Ans at the back: 3.9kN/m

My ans: 3.5kN/m
 
Last edited:
Physics news on Phys.org
  • #2
I assume you are to model this as simple harmonic motion of the trampolinist, in which case the book answer seems correct. Show how you arrived at your answer.
 
  • #3
Doc Al said:
I assume you are to model this as simple harmonic motion of the trampolinist, in which case the book answer seems correct. Show how you arrived at your answer.
i work out the period by taking 80 divide by 60s which gives 1.33Hz
T = 1/f therefore T = 0.75s

using T = 2pi (square root m/k)
0.75 = 2pi (square root 50kg/k)
0.5625 = 39.488656 x 50/k
k = 3510 N/m
 
Last edited:
  • #4
3.9 kN/m2? A spring constant is in "force per distance", kN/m, not m^2.
 
  • #5
So far, so good. Now check your arithmetic.

(Yeah, those units are wacked. Good catch by Halls.)
 
  • #6
HallsofIvy said:
3.9 kN/m2? A spring constant is in "force per distance", kN/m, not m^2.
edited...thanks for telling :smile:
 
  • #7
Doc Al said:
So far, so good. Now check your arithmetic.

(Yeah, those units are wacked. Good catch by Halls.)
D'OH! ur rite...i mixed up that qn with the one above it ...this one has 55kg not 50kg...

i wonder if there is a smiley for "bashing my own head with textbook"...
 

Related to Confirm Physics Textbook Error: Trampolinist Spring Constant

1. What is a spring constant?

A spring constant is a physical quantity that represents the stiffness of a spring. It is a measure of how much force is required to stretch or compress a spring by a certain distance.

2. How is a spring constant calculated?

The spring constant is calculated by dividing the applied force by the displacement of the spring. It can also be calculated by dividing the potential energy stored in a spring by the amount of stretch or compression.

3. What is the significance of the spring constant in physics?

The spring constant is an important concept in physics because it helps us understand the relationship between force and displacement in a spring. It also plays a role in many other areas of physics, such as oscillations and harmonic motion.

4. How does the spring constant affect the behavior of a trampolinist?

The spring constant of a trampoline determines how much force is required to stretch the trampoline's surface and how quickly it will return to its original shape. A higher spring constant will result in a stiffer trampoline, while a lower spring constant will result in a bouncier trampoline.

5. Why is it important to confirm any errors in a physics textbook?

Confirming errors in a physics textbook is important because it ensures the accuracy of the information being taught to students. Inaccurate information can lead to misunderstandings and incorrect application of concepts, which can be detrimental in the study and practice of physics.

Similar threads

  • Introductory Physics Homework Help
Replies
18
Views
920
  • Introductory Physics Homework Help
Replies
3
Views
423
  • Introductory Physics Homework Help
Replies
7
Views
908
  • Introductory Physics Homework Help
Replies
11
Views
4K
  • Introductory Physics Homework Help
Replies
1
Views
2K
Replies
3
Views
858
Replies
3
Views
1K
  • Introductory Physics Homework Help
Replies
9
Views
2K
  • Science and Math Textbooks
Replies
19
Views
1K
  • Introductory Physics Homework Help
Replies
7
Views
14K
Back
Top